upsc 2019 paper...upsc 2019 paper set- d 1. with reference to asian infrastructure investment bank...

98

Upload: others

Post on 28-Jun-2021

5 views

Category:

Documents


0 download

TRANSCRIPT

Page 1: UPSC 2019 PAPER...UPSC 2019 PAPER SET- D 1. With reference to Asian Infrastructure Investment Bank (AIIB), consider the following statements 1. AIIB has more than 80 member nations
Page 2: UPSC 2019 PAPER...UPSC 2019 PAPER SET- D 1. With reference to Asian Infrastructure Investment Bank (AIIB), consider the following statements 1. AIIB has more than 80 member nations

UPSC 2019 PAPER

SET- D

1. With reference to Asian Infrastructure Investment Bank (AIIB), consider the following

statements

1. AIIB has more than 80 member nations.

2. India is the largest shareholder in AIIB.

3. AIIB does not have any members from outside Asia.

Which of the statements given above is / are correct?

(a) 1 only

(b) 2 and 3 only

(c) 1 and 3 only

(d) 1, 2 and 3

Sol. 1 (a) 1 only

Source: PrepMate International Organizations and Bilateral Relations, Chapter 4, Page 96

Statement 1 is correct.

Statements 2 and 3 are incorrect.

2. What was the purpose of Inter-Creditor Agreement signed by Indian banks and financial

institutions recently?

(a) To lessen the Government of India's perennial burden of fiscal deficit and current

account deficit

(b) To support the infrastructure projects of Central and State Governments

Page 3: UPSC 2019 PAPER...UPSC 2019 PAPER SET- D 1. With reference to Asian Infrastructure Investment Bank (AIIB), consider the following statements 1. AIIB has more than 80 member nations

(c) To act as independent regulator in case of applications for loans of Rs. 50 crore or more

(d) To aim at faster resolution of stressed assets of Rs. 50 crore or more which are-under

consortium lending

Sol. 2 (d) To aim at faster resolution of stressed assets of Rs. 50 crore or more which are-

under consortium lending

Source: PrepMate Current Affairs

3. The Chairman of public sector banks are selected by the

(a) Banks Board Bureau

(b) Reserve Bank of India

(c) Union Ministry of Finance

(d) Management of concerned bank

Sol. 3 (a) Banks Board Bureau

Source: PrepMate Current Affairs

Page 4: UPSC 2019 PAPER...UPSC 2019 PAPER SET- D 1. With reference to Asian Infrastructure Investment Bank (AIIB), consider the following statements 1. AIIB has more than 80 member nations

It is to be noted that Banks Board Bureau recommends the appointment of a particular

person as Chairman of a public sector bank. Thus, Chairman of public sector banks is

selected by Banks Board Bureau. However, the final appointment is made by Union

Government.

4. Consider the following statements:

1. Petroleum and Natural Gas Regulatory Board (PNGRB) is the first regulatory body set up

by the Government of India.

2. One of the tasks of PNGRB is to ensure competitive markets for gas.

3. Appeals against the decisions of PNGRB go before the Appellate Tribunals for Electricity.

Which of the statements given above are correct?

(a) 1 and 2 only

Page 5: UPSC 2019 PAPER...UPSC 2019 PAPER SET- D 1. With reference to Asian Infrastructure Investment Bank (AIIB), consider the following statements 1. AIIB has more than 80 member nations

(b) 2 and 3 only

(c) 1 and 3 only

(d) 1, 2 and 3

Sol. 4 (b) 2 and 3 only

Source: PrepMate Current affairs

Statement 1 is incorrect. There are many regulatory bodies in India which are working

since independence. Some examples of these bodies are Reserve Bank of India, Railway

board, etc.

Once we come to know that statement 1 is incorrect. We can eliminate all the answer

choices which have statement 1 in them. Thus, we are left with option (b) only.

5. With reference to communication technologies, what is/are the difference / differences

between LTE (Long-Term Evolution) and VoLTE (Voice over Long-Term Evolution)?

1. LTE is commonly marketed as 3G and VoLTE is commonly marketed as advanced 3G.

2. LTE is data-only technology and VoLTE is voice-only technology.

Select the correct answer using the code given below.

(a) 1 only

(b) 2 only

(c) Both 1 and 2

(d) Neither 1 nor 2

Sol.5 (d) Neither 1 nor 2

Source: PrepMate Science & Technology, Chapter 13 Communication Technology, Page 190

Both the Statements are incorrect.

Page 6: UPSC 2019 PAPER...UPSC 2019 PAPER SET- D 1. With reference to Asian Infrastructure Investment Bank (AIIB), consider the following statements 1. AIIB has more than 80 member nations

6. Which of the following statements is / are correct regarding the Maternity Benefit

(Amendment) Act, 2017?

1. Pregnant women are entitled for three months pre-delivery and three months post-

delivery paid leave.

2. Enterprises with creches must allow the mother minimum six creche visits daily.

3. Women with two children get reduced entitlements.

Select the correct answer using the code given below.

(a) 1 and 2 only

(b) 2 only

(c) 3 only

(d) 1, 2 and 3

Sol.6 (c) 3 only

Source: PrepMate Current Affairs

Statements 1 and 2 are incorrect. Only Statement 3 is correct.

Page 7: UPSC 2019 PAPER...UPSC 2019 PAPER SET- D 1. With reference to Asian Infrastructure Investment Bank (AIIB), consider the following statements 1. AIIB has more than 80 member nations

7. Which one of the following is not a sub-index of the World Bank's 'Ease of Doing

Business Index'?

(a) Maintenance of law and order

(b) Paying taxes

(c) Registering property

(d) Dealing with construction permits

Sol. 7 (a) Maintenance of law and order

Source: PrepMate International Organizations and Bilateral Relations, Chapter 7, Page 167

8. In India, 'extended producer responsibility' was introduced as an important feature in

which of the following?

(a) The Bio-medical Waste (Management and Handling) Rules, 1998

(b) The Recycled plastic (Manufacturing and Usage) Rules, 1999

(c) The e-Waste (Management and Handling) Rules, 2011

Page 8: UPSC 2019 PAPER...UPSC 2019 PAPER SET- D 1. With reference to Asian Infrastructure Investment Bank (AIIB), consider the following statements 1. AIIB has more than 80 member nations

(d) The Food Safety and Standard Regulations, 2011

Sol. 8 (c) The e-Waste (Management and Handling) Rules, 2011

Source: PrepMate Environment & Biodiversity, Chapter 5, Page 90

9. The economic cost of food grains to the Food Corporation of India is Minimum Support

Price and bonus (if any) paid to the farmers plus

(a) transportation cost only

(b) interest cost only

(c) procurement incidentals and distribution cost

(d) procurement incidentals and charges for godowns

Sol. 9 (c) procurement incidentals and distribution cost

Source: PrepMate Economics, Chapter 7, Page 126

10. In the context of any country, which one of the following would be considered as part of

its social capital?

(a) The proportion of literates in the population

(b) The stock of its buildings, other infrastructure and machines

Page 9: UPSC 2019 PAPER...UPSC 2019 PAPER SET- D 1. With reference to Asian Infrastructure Investment Bank (AIIB), consider the following statements 1. AIIB has more than 80 member nations

(c) The size of population in the working age group

(d) The level of mutual trust and harmony in the society

Sol. 10 (d) The level of mutual trust and harmony in the society

Topic: Miscellaneous

This question is unexpected. However, UPSC does have tendency to ask such questions.

Social capital broadly refers to those factors which effect functioning of social groups. Such

factors include interpersonal relationships, a shared sense of identity, a shared

understanding, shared norms, shared values, trust, cooperation, and reciprocity. Thus,

option (d) is the right answer.

This question is not covered in PrepMate sources.

11. The Service Area Approach was implemented under the purview of

(a) Integrated Rural Programme

(b) Lead Bank Scheme

(c) Mahatma Gandhi National Rural Employment Guarantee Scheme

(d) National Skill Development Mission

Sol. 11 (b) Lead Bank Scheme

Source: Economics book, Chapter 11, Page 175

12. With reference to the management of minor minerals in India, consider the following

statements:

1. Sand is a 'minor mineral' according to the prevailing law in the country.

2. State Governments have the power to grant mining leases of minor minerals, but the

powers regarding the formation of rules related to the grant of minor minerals lie with the

Central Government.

Page 10: UPSC 2019 PAPER...UPSC 2019 PAPER SET- D 1. With reference to Asian Infrastructure Investment Bank (AIIB), consider the following statements 1. AIIB has more than 80 member nations

3. State Governments have the power to frame rules to prevent illegal mining of minor

minerals.

Which of the statements given above is / are correct?

(a) 1 and 3 only

(b) 2 and 3 only

(c) 3 only

(d) 1, 2 and 3

Sol. 12 (a) 1 and 3 only

Topic: Miscellaneous

Statement 1 is correct. Stand is a mirror mineral, as defined under section 3(e) of the Mines

and Minerals (Development and Regulation) Act, 1957 (MMDR Act).

Statement 2 is incorrect. Section 15 of the MMDR Act empowers state governments to

make rules for regulating the grant of mineral concessions in respect of minor minerals.

The regulation of grant of mineral concessions for minor minerals is, therefore, within the

purview of the state governments.

Statement 3 is correct. Section 23C of the MMDR Act, 1957 empowers state government to

frame rules to prevent illegal mining, transportation and storage of minerals.

13. Consider the following statements:

1. Most of India's external debt is owed by governmental entities.

2. All of India's external debt is denominated in US dollars.

Which of the statements given above is / are correct?

(a) 1 only

(b) 2 only

(c) Both 1 and 2

(d) Neither 1 nor 2

Sol. 13 (d) Neither 1 nor 2

Source: PrepMate Economics Book, Chapter 23, Page 325

Statement 1 is incorrect: Out of India’s total external debt, nearly 20% is owed by

governmental entities.

Page 11: UPSC 2019 PAPER...UPSC 2019 PAPER SET- D 1. With reference to Asian Infrastructure Investment Bank (AIIB), consider the following statements 1. AIIB has more than 80 member nations

Statement 2 is incorrect. India’s External debt is denominated in multiple currencies. As on

31st December, 2017, 48.2% of debt was denominated in US dollars. The rest of debt is

denominated in Indian Rupee (for instance- masala bonds), Special Drawing rights,

Japanese Yen, Euro and other currencies.

14. Which of the following is not included in the assets of a commercial bank in India?

(a) Advances

(b) Deposits

(c) Investments

(d) Money at call and short notice

Sol. 14 (b) Deposits

Source: Economics book, Chapter 3, Page 59

15. In the context of India, which of the following factors is/are contributor/ contributors

to reducing the risk of a currency crisis?

1. The foreign currency earnings of India's IT sector

2. Increasing the government expenditure

3. Remittances from Indians abroad

Page 12: UPSC 2019 PAPER...UPSC 2019 PAPER SET- D 1. With reference to Asian Infrastructure Investment Bank (AIIB), consider the following statements 1. AIIB has more than 80 member nations

Select the correct answer using the code given below.

(a) 1 only

(b) 1 and 3 only

(c) 2 only

(d) 1, 2 and 3

Sol. 15 (b) 1 and 3 only

Source: PrepMate Economics book, Chapter 21, Page 306

This question is based on understanding of basic economics and knowledge of current

affairs. Currency crisis in the present context refers to depreciation of rupee. Any foreign

exchange transaction which generates demand of rupee can help in arresting the

depreciation of rupee.

Statement 1 is correct. Foreign currency earnings of India's IT sector generate supply of

dollar and demand of Indian rupee. Thus, this transaction helps in arresting the

depreciation of rupee.

Statement 2 is incorrect. Increase in government expenditure doesn’t directly affects the

demand and supply of rupee.

Statement 3 is correct. Remittances from Indians abroad generate supply of dollar and

demand of Indian rupee. Thus, this transaction helps in arresting the depreciation of rupee.

Thus, Statements 1 and 3 are correct.

Page 13: UPSC 2019 PAPER...UPSC 2019 PAPER SET- D 1. With reference to Asian Infrastructure Investment Bank (AIIB), consider the following statements 1. AIIB has more than 80 member nations

16. Which one of the following suggested that the Governor should be an eminent person

from outside the State and should be a detached figure without intense political links or

should not have taken part in politics in the recent past?

(a) First Administrative Reforms Commission (1966)

(b) Rajamannar Committee (1969)

(c) Sarkaria Commission (1983)

(d) National Commission to Review the Working of the Constitution (2000)

Sol. 16 (c) Sarkaria Commission (1983)

Source: PrepMate Current Affairs

There have been instances in recent past when Governors have acted in the interest of

central government. It is usual for news articles to recall the recommendations of Sarkaria

commission on office of governor at such instances. There was one article in Indian Express

which was shared in PrepMate newsjuice. Relevant extracts of that article are pasted

below.

17. Which of the following is issued by registered foreign portfolio investors to overseas

investors who want to be part of the Indian stock market without registering themselves

directly?

(a) Certificate of Deposit

(b) Commercial Paper

Page 14: UPSC 2019 PAPER...UPSC 2019 PAPER SET- D 1. With reference to Asian Infrastructure Investment Bank (AIIB), consider the following statements 1. AIIB has more than 80 member nations

(c) Promissory Note

(d) Participatory Note

Sol. 17 (d) Participatory Note

Source: PrepMate Economics, Chapter 24, Page 343

There is a direct question in practice set given at end of chapter which is similar to this

question. A participatory note, commonly known as a P-note or PN, is a certificate issued by

a registered foreign institutional investor (FII) to an overseas investor who invests in

Indian stock markets. FII is registered with SEBI directly, but overseas investor is not

required to register themselves with SEBI. Thus, overseas investor invests in Indian stock

market through a registered foreign institutional investor. As proof of investment, a

certificate is issued by FII to the overseas investor.

18. Consider the following statements:

1. As per law, the Compensatory Afforestation Fund Management and Planning Authority

exists at both National and State levels.

2. People's participation is mandatory in the compensatory afforestation programmes

carried out under the Compensatory Afforestation Fund Act, 2016.

Which of the statements given above is / are correct?

(a) 1 only

(b) 2 only

(c) Both 1 and 2

(d) Neither 1 nor 2

Sol. 18 (a) 1 only

Source: PrepMate Current Affairs

Statement 1 is correct.

Page 15: UPSC 2019 PAPER...UPSC 2019 PAPER SET- D 1. With reference to Asian Infrastructure Investment Bank (AIIB), consider the following statements 1. AIIB has more than 80 member nations

Statement 2 is incorrect. Compensatory Afforestation Fund Act, 2016 was criticized on the

grounds that it does not provide for people’s participation in afforestation programmes.

19. In India, which of the following review the independent regulators in sectors like

telecommunications, insurance, electricity, etc.?

1. Ad Hoc Committees set up by the Parliament

2. Parliamentary Department Related Standing Committees

3. Finance Commission

4. Financial Sector Legislative Reforms Commission

5. NITI Aayog

Select the correct answer using the code given below.

(a) 1 and 2

(b) 1, 3 and 4

(c) 3, 4 and 5

(d) 2 and 5

Sol.19 (a) 1 and 2

Source: PrepMate Polity book, Chapter 8, Page 143 and 144

This is an indirect question. This question requires understanding about the functioning of

the mentioned bodies.

Page 16: UPSC 2019 PAPER...UPSC 2019 PAPER SET- D 1. With reference to Asian Infrastructure Investment Bank (AIIB), consider the following statements 1. AIIB has more than 80 member nations

Statement 1 is correct: Ad Hoc Committees of parliament can be established to advice and

enquire on particular matter. Thus, they can be established to review the independent

regulators.

Statement 2 is correct: Parliamentary Department Related Standing Committees undertake

review of functioning, accounts and bills of particular ministry/ department. Thus, they can

also review the independent regulator pertaining to that particular department.

Statement 3 is incorrect: Finance commission recommends division of funds between

Centre and States. It does not review the independent regulators.

Statement 4 is incorrect: Financial Sector Legislative Reforms Commission make

recommendations to bring reforms in financial sector.

Statement 5 is incorrect: Niti Aayog has undertaken various tasks such as adoption of best

practices in government machinery, oversee performance of certain tasks,

recommendations on certain issues, etc. However, it is not involved in review of

independent regulators.

20. With reference to India's Five-Year Plans, which of the following statements is/are

correct?

1. From the Second Five-Year Plan, there was a determined thrust towards substitution of

basic and capital good industries.

Page 17: UPSC 2019 PAPER...UPSC 2019 PAPER SET- D 1. With reference to Asian Infrastructure Investment Bank (AIIB), consider the following statements 1. AIIB has more than 80 member nations

2. The Fourth Five-Year Plan adopted the objective of correcting the earlier trend of

increased concentration of wealth and economic power.

3. In the Fifth Five-Year Plan, for the first time, the financial sector was included as an

integral part of the Plan.

Select the correct answer using the code given below.

(a) 1 and 2 only

(b) 2 only

(c) 3 only

(d) 1, 2 and 3

Sol. 20 (a) 1 and 2 only

Source: PrepMate Economics, Chapter 13, Page 192

This question requires not just understanding of Planning and but also keen analysis.

Statement 1 is correct: The thrust towards substitution of basic and capital good industries

is from second five year plan after adoption of Mahalanobis model.

Statement 2 is correct: In order to correct the trend of increased concentration of wealth

and economic power, government carried out nationalization of important industries such

as banking, insurance, etc.

Page 18: UPSC 2019 PAPER...UPSC 2019 PAPER SET- D 1. With reference to Asian Infrastructure Investment Bank (AIIB), consider the following statements 1. AIIB has more than 80 member nations

Statement 3 is incorrect: Financial sector includes industries such as banking, insurance

which were included from the first plan itself.

21. With reference to the Constitution of India, consider the following statements:

1. No High Court shall have the jurisdiction to declare any central law to be constitutionally

invalid.

2. An amendment to the Constitution of India cannot be called into question by the

Supreme Court of India.

Which of the statements given above is / are correct?

(a) 1 only

(b) 2 only

(c) Both 1 and 2

(d) Neither 1 nor 2

Sol. 21 (d) Neither 1 nor 2

This question can be answered by reading either PrepMate Indian Polity, Chapter 5, Page

43 or PrepMate Current affairs.

Statement 1 is incorrect: The Indian Constitution has established an integrated judicial

system with the Supreme Court at the top and the state high courts below it. This single

system of courts enforces and reviews both the Central laws as well as the state laws. Thus,

Courts including High Courts have the jurisdiction to declare any central law to be

constitutionally invalid.

Alternately, there have been many instances in recent times when high courts have

declared central law as constitutionally invalid.

Page 19: UPSC 2019 PAPER...UPSC 2019 PAPER SET- D 1. With reference to Asian Infrastructure Investment Bank (AIIB), consider the following statements 1. AIIB has more than 80 member nations

Statement 2 is also incorrect: Supreme Court held in the Kesavananda Bharati case (1973)

that a Constitutional amendment can be challenged on the ground that it violates the ‘basic

structure’ of the Constitution and hence, can be declared as void.

Source: PrepMate Polity, Chapter 5, Page 43

Even recently, Supreme Court has held 99th Constitutional amendment act which provided

for NJAC as unconstitutional and void.

Source: PrepMate Polity, Chapter 8, Page 150

Page 20: UPSC 2019 PAPER...UPSC 2019 PAPER SET- D 1. With reference to Asian Infrastructure Investment Bank (AIIB), consider the following statements 1. AIIB has more than 80 member nations

22. Consider the following statements:

1. Purchasing Power Parity (PPP) exchange rates are calculated by comparing the prices of

the same basket of goods and services in different countries.

2. In terms of PPP dollars, India is the sixth largest economy in the world.

Which of the statements given above is / are correct?

(a) 1 only

(b) 2 only

(c) Both 1 and 2

(d) Neither 1 nor 2

Sol. 22 (a) 1 only

Source: PrepMate Economics, Chapter 1, Page 2

Page 21: UPSC 2019 PAPER...UPSC 2019 PAPER SET- D 1. With reference to Asian Infrastructure Investment Bank (AIIB), consider the following statements 1. AIIB has more than 80 member nations

23. With reference to the cultivation of Kharif crops in India in the last five years, consider

the following statements:

1. Area under rice cultivation is the highest.

2. Area under the cultivation of jowar is more than that of oilseeds.

3. Area of cotton cultivation is more than that of sugarcane.

4. Area under sugarcane cultivation has steadily decreased.

Which of the statements given above are correct?

(a) 1 and 3 only

(b) 2, 3 and 4 only

(c) 2 and 4 only

(d) 1, 2, 3 and 4

Sol. 23 (a) 1 and 3 only

Page 22: UPSC 2019 PAPER...UPSC 2019 PAPER SET- D 1. With reference to Asian Infrastructure Investment Bank (AIIB), consider the following statements 1. AIIB has more than 80 member nations

Source: Geography book, Chapter 27, Page 357

This question can be solved easily through combination of selection and elimination

techniques. The question stem (With reference to the cultivation of Kharif crops in India in

the last five years, consider the following statements) makes it even easier to solve this

question. The question talks about only kharif crops.

Let us start from Statement 1. It is clear that amongst various kharif crops, the largest area

is under rice cultivation.

Once we ascertain that Statement 1 is correct, we are left with answer options (a) 1 and 3

only and

(d) 1, 2, 3 and 4.

Now, we can safely rule out Statement 4. There is no information that area under sugarcane

cultivation has steadily decreased. It would have been a big news that area under

sugarcane cultivation is continuously decreasing.

Page 23: UPSC 2019 PAPER...UPSC 2019 PAPER SET- D 1. With reference to Asian Infrastructure Investment Bank (AIIB), consider the following statements 1. AIIB has more than 80 member nations

Thus, we can safely eliminate option (d). Our answer is remaining option, which is option

(a).

There is another interesting statement in this question, Statement 3- Area of cotton

cultivation is more than that of sugarcane. Sugarcane is cultivated both as kharif and rabi

crop. On the other hand, cotton is cultivated only as kharif crop. Certainly, when we

consider this fact, then for sure area of cotton cultivation is more than that of sugarcane.

24. Among the agricultural commodities imported by India, which one of the following

accounts for the highest imports in terms of value in the last five years?

(a) Spices

(b) Fresh fruits

(c) Pulses

(d) Vegetable oils

Sol. 24 (d) Vegetable oils

Source: PrepMate Economics, Chapter 27, Page 382

Among the agricultural commodities imported by India, Vegetable oils constitute

significant chunk of imports.

25. In the context of polity, which one of the following would you accept as the most

appropriate definition of liberty?

(a) Protection against the tyranny of political rulers

(b) Absence of restraint

Page 24: UPSC 2019 PAPER...UPSC 2019 PAPER SET- D 1. With reference to Asian Infrastructure Investment Bank (AIIB), consider the following statements 1. AIIB has more than 80 member nations

(c) Opportunity to do whatever one likes

(d) Opportunity to develop oneself fully

Sol. 25 (d) Opportunity to develop oneself fully

Topic: Polity

There has been question on concept of liberty in prelims 2017 and 2018 as well. PrepMate

Polity book contains detailed explanations to these questions. These explanations and

understanding of fundamental rights are useful to solve the above question. However, we

don’t claim this question to be appearing from our study material because this question

demands deeper theoretical knowledge of concepts. Let us examine the answer options.

Option (a) is incorrect: It fails to express the complete idea of liberty. Protection against the

tyranny of political rulers does not express complete idea of liberty. Liberty not only

confers protection but it also grants multifaceted freedoms.

Option (b) is incorrect: If there is no restraint, then the actions of people may erode other’s

liberty. Thus, restraint is essential to enjoy liberty. For instance, even fundamental rights

have restrictions. There are 8 limitations on freedom of speech and expression.

Option (c) is incorrect: It also attempts to grant freedom without any restraint. The

explanation given for proving option (b) as incorrect, is also applicable for option (c).

Option (d) is correct: Liberty (with restraints) provide us with opportunity to develop fully

in life. The development may be in different dimensions such as social, economic,

intellectual, etc. Let us look at the other side. If there were severe restrictions by state on

economic activity, then we may not be able to pursue our economic upliftment. It is to be

noted that a developed individual has utmost respect for restraints because one’s restraint

enable other people to enjoy their own liberty.

26. Which one of the following is not the most likely measure the Government/RBI takes to

stop the slide of Indian rupee?

(a) Curbing imports of non-essential goods and promoting exports

(b) Encouraging Indian borrowers to issue rupee denominated Masala Bonds

(c) Easing conditions relating to external commercial borrowing

(d) Following an expansionary monetary policy

Sol. 26 (d) Following an expansionary monetary policy

Source: PrepMate Economics book, Chapter 21, Page 306 and Chapter 24, Page 345

Options (a), (b) and (c) are adopted to curb the slide of Indian rupee.

Page 25: UPSC 2019 PAPER...UPSC 2019 PAPER SET- D 1. With reference to Asian Infrastructure Investment Bank (AIIB), consider the following statements 1. AIIB has more than 80 member nations

Chapter 21, Page 306

Option (c) is not the answer. Easing conditions relating to external commercial borrowing

will encourage fresh loans from external sources.

Option (b) is not the answer.

Source PrepMate Economics book, Chapter 24, Page 345

Page 26: UPSC 2019 PAPER...UPSC 2019 PAPER SET- D 1. With reference to Asian Infrastructure Investment Bank (AIIB), consider the following statements 1. AIIB has more than 80 member nations

Option (d) is the answer. Option (d) will not help in curbing the slide of Indian rupee.

Rather, it may even further reduce the value of Indian rupee. If expansionary monetary

policy is followed, it leads to increase in money supply. Increase in money supply may lead

to inflation, which may further lead to fall in value of rupee.

27. Consider the following statements:

The Reserve Bank of India's recent directives relating to 'Storage of Payment System Data',

popularly known as data diktat, command the payment system providers that

1. they shall ensure that entire data relating to payment systems operated by them are

stored in a system only in India

2. they shall ensure that the systems are owned and operated by public sector enterprises

3. they shall submit the consolidated system audit report to the Comptroller and Auditor

General of India by the end of the calendar year

Which of the statements given above is/are correct?

(a) 1 only

(b) 1 and 2 only

(c) 3 only

Page 27: UPSC 2019 PAPER...UPSC 2019 PAPER SET- D 1. With reference to Asian Infrastructure Investment Bank (AIIB), consider the following statements 1. AIIB has more than 80 member nations

(d) 1, 2 and 3

Sol. 27 (a) 1 only

Topic: Current Affairs

Statement 1 is correct: It is important to have unfettered supervisory access to data

relating to payment systems operating in India. This access is only possible if data is stored

in India. This directive was released by the RBI.

RBI directive was as follows. All system providers shall ensure that the entire data relating

to payment systems operated by them are stored in a system only in India. This data should

include the full end-to-end transaction details / information collected / carried / processed

as part of the message / payment instruction. For the foreign leg of the transaction, if any,

the data can also be stored in the foreign country, if required.

Statement 2 is not correct: There was no directive related to the ownership and operation

of the systems by public sector enterprises.

Statement 3 is not correct: System providers shall submit the System Audit Report (SAR)

conducted by CERT-IN empaneled auditors. The SAR duly approved by the Board of the

system providers should be submitted to the Reserve Bank by the end of calendar year.

28. Which of the following adopted a law on data protection and privacy for its citizens

known as 'General Data Protection Regulation' in April 2016 and started implementation of

it from 25th May, 2018?

(a) Australia

(b) Canada

(c) The European Union

(d) The United States of America

Sol. 28 (c) The European Union

Source: PrepMate Current Affairs

Page 28: UPSC 2019 PAPER...UPSC 2019 PAPER SET- D 1. With reference to Asian Infrastructure Investment Bank (AIIB), consider the following statements 1. AIIB has more than 80 member nations

29. Recently, India signed a deal known as 'Action Plan for Prioritization and

Implementation of Cooperation Areas in the Nuclear Field' with which of the following

countries?

(a) Japan

(b) Russia

(c) The United Kingdom

(d) The United States of America

Sol. 29(b) Russia

Topic: Current Affairs

This is a tough question. The deal asked in the question is related to action plan only and

not the actual agreement on cooperation in nuclear field. Usually, there are many such

bilateral deals with other nations. They are in nature of Memorandum of agreement and

not the actual agreement.

Action Plan for Prioritization and Implementation of Co-operation Areas in the Nuclear

Field Identified Jointly by India and Russia was signed by India with Russia. It was signed

on 5th October, 2018 in New Delhi during the visit of Vladimir Putin, President of the

Russian Federation to India.

30. The money multiplier in an economy increases with which one of the following?

(a) Increase in the cash reserve ratio

(b) Increase in the banking habit of the population

Page 29: UPSC 2019 PAPER...UPSC 2019 PAPER SET- D 1. With reference to Asian Infrastructure Investment Bank (AIIB), consider the following statements 1. AIIB has more than 80 member nations

(c) Increase in the statutory liquidity ratio

(d) Increase in the population of the country

Sol. 30 (b) Increase in the banking habit of the population

Source: PrepMate Economics, Chapter 3, Page 61

Options (a) and (c) are incorrect. If cash reserve ratio and statutory liquidity ratio

reduce (and not increase), then money multiplier increases. Reserve money, which is

denominator for calculating money multiplier, includes statutory reserves of banks held

with themselves and cash reserves of banks held with the RBI. The lower the value of

denominator in a fraction, the higher the value of overall fraction.

Option (d) is not directly related to the question.

Option (c) is the correct answer. M1 and M3, which are numerator for calculating money

multiplier, include demand deposits and total deposits by public. The amount of deposits

made by people in banks also depends upon the banking habit of the population. Moreover,

greater the value of numerator, greater is the value of overall fraction. Thus, increase in the

banking habit of the population, increases the money multiplier in the economy.

Page 30: UPSC 2019 PAPER...UPSC 2019 PAPER SET- D 1. With reference to Asian Infrastructure Investment Bank (AIIB), consider the following statements 1. AIIB has more than 80 member nations

31. Consider the following statements about Particularly Vulnerable Tribal Groups (PVTGs)

in India:

1. PVTGs reside in 18 States and one Union Territory.

2. A stagnant or declining population is one of the criteria for determining PVTG status.

3. There are 95 PVTGs officially notified in the country so far.

4. Irular and Konda Reddi tribes are included in the list of PVTGs.

Which of the statements given above are correct?

(a) 1, 2 and 3

(b) 2, 3 and 4

(c) 1, 2 and 4

(d) 1, 3 and 4

Sol. 31 (c) 1, 2 and 4

Source: PrepMate Geography Book, Chapter 30, Page 401

Statement 3 is incorrect. Once we rule out statement 3, answer is obvious.

Page 31: UPSC 2019 PAPER...UPSC 2019 PAPER SET- D 1. With reference to Asian Infrastructure Investment Bank (AIIB), consider the following statements 1. AIIB has more than 80 member nations

32. With reference to the Constitution of India, prohibitions or limitations or provisions

contained in ordinary laws cannot act as prohibitions or limitations on the constitutional

powers under Article 142. It could mean which one of the following?

(a) The decisions taken by the Election Commission of India while discharging its duties

cannot be challenged in any court of law.

(b) The Supreme Court of India is not constrained in the exercise of its powers by laws

made by the Parliament.

(c) In the event of grave financial crisis in the country, the President of India can declare

Financial Emergency without the counsel from the Cabinet.

(d) State Legislatures cannot make laws on certain matters without the concurrence of

Union Legislature.

Sol. 32 (b) The Supreme Court of India is not constrained in the exercise of its powers by

laws made by the Parliament.

Source: PrepMate Polity, Chapter 8, Page 153

This is an interesting question. This question requires combined analysis of Article 142 and

information given in question stem. Article 142 grants extraordinary powers to the

Supreme Court to do complete justice. For exercising powers conferred by Article 142, the

SC is not bound by any law made by Parliament.

33. With reference to the Legislative Assembly of a State in India, consider the following

statements:

1. The Governor makes a customary address to Members of the House at the

commencement of the first session of the year.

2. When a State Legislature does' not have a rule on a particular matter, it follows the Lok

Sabha rule on that matter.

Which of the statements given above is / are correct?

Page 32: UPSC 2019 PAPER...UPSC 2019 PAPER SET- D 1. With reference to Asian Infrastructure Investment Bank (AIIB), consider the following statements 1. AIIB has more than 80 member nations

(a) 1 only

(b) 2 only

(c) Both 1 and 2

(d) Neither 1 nor 2

Sol. 33 (a) 1 only

Source of 1st statement: PrepMate Polity, Chapter 8, Page 128 and Chapter 9, Page 173

Statement 2 is incorrect. Article 208 Rules of procedure

(1) A House of the Legislature of a State may make rules for regulating subject to the

provisions of this Constitution, its procedure and the conduct of its business

(2) Until rules are made under clause (1), the rules of procedure and standing orders in

force immediately before the commencement of this Constitution with respect to the

Legislature for the corresponding Province shall have effect in relation to the Legislature of

the State subject to such modifications and adaptations as may be made therein by the

Speaker of the Legislative Assembly, or the Chairman of the Legislative Council, as the case

may be.

34. Consider the following statements:

1. The United Nations Convention against Corruption (UNCAC) has a 'Protocol against the

Smuggling of Migrants by Land, Sea and Air'.

2. The UNCAC is the ever-first legally binding global anti-corruption instrument.

3. A highlight of the United Nations Convention against Transnational Organized Crime

(UNTOC) is the inclusion of a specific chapter aimed at returning assets to their rightful

owners from whom they had been taken illicitly.

Page 33: UPSC 2019 PAPER...UPSC 2019 PAPER SET- D 1. With reference to Asian Infrastructure Investment Bank (AIIB), consider the following statements 1. AIIB has more than 80 member nations

4. The United Nations Office on Drugs and Crime (UNODC) is mandated by its member

States to assist in the implementation of both UNCAC and UNTOC.

Which of the statements given above are correct?

(a) 1 and 3 only

(b) 2, 3 and 4 only

(c) 2 and 4 only

(d) 1, 2, 3 and 4

Sol. 34 (c) 2 and 4 only

Source: PrepMate International Organizations and Bilateral Relations, Chapter 1, Page 15

Statement 4 is correct.

Statement 1 is incorrect. United Nations Convention against Corruption (UNCAC) is against

corruption and is not related to Smuggling of Migrants. UNCAC has been in news in context

of amendments in Prevention of Corruption Act to make it in line with UNCAC. UNTOC has

a 'Protocol against the Smuggling of Migrants by Land, Sea and Air'.

Once we know that statement 1 is incorrect and statement 4 is correct, we are left with

options (b) and (c).

Statement 3 is also incorrect. UNTOC has three supplementary protocols namely

Trafficking in Persons, especially Women and Children; Smuggling of Migrants; and illicit

Manufacturing of and Trafficking in Firearms. It does not have specific chapter aimed at

returning assets to their rightful owners from whom they had been taken illicitly.

35. Consider the following statements:

1. As per recent amendment to the Indian Forest Act, 1927, forest dwellers have the right to

fell the bamboos grown on forest areas.

2. As per the Scheduled Tribes and Other Traditional Forest Dwellers (Recognition of

Forest Rights) Act, 2006, bamboo is a minor forest produce.

Page 34: UPSC 2019 PAPER...UPSC 2019 PAPER SET- D 1. With reference to Asian Infrastructure Investment Bank (AIIB), consider the following statements 1. AIIB has more than 80 member nations

3. The Scheduled Tribes and Other Traditional Forest Dwellers (Recognition of Forest

Rights) Act, 2006 allows ownership of minor forest produce to forest dwellers.

Which of the statements given above is / are correct?

(a) 1 and 2 only

(b) 2 and 3 only

(c) 3 only

(d) 1, 2 and 3

Sol. 35 (b) 2 and 3 only

Source: PrepMate Current Affairs have covered Statements 1 and 3 but not Statement 2.

Statement 1 is incorrect. The Indian Forest Act, 1927, was amended to permit felling of

bamboo grown in non-forest areas.

Once we identify that Statement 1 is incorrect. We are left with two options- (b) 2 and 3

only and (c) 3 only.

Statement 3 is correct: The Scheduled Tribes and Other Traditional Forest Dwellers

(Recognition of Forest Rights) Act, 2006 provides ownership of minor forest produce to

forest dwellers.

Page 35: UPSC 2019 PAPER...UPSC 2019 PAPER SET- D 1. With reference to Asian Infrastructure Investment Bank (AIIB), consider the following statements 1. AIIB has more than 80 member nations

Statement 2 is correct. According to the Act, ""minor forest produce"" includes all non-

timber forest produce of plant origin including bamboo, brush wood, stumps, cane, tussar,

cocoons, honey, wax, lac, tendu or kendu leaves, medicinal plants and herbs, roots, tubers

and the like.

36. Which Article of the Constitution of India safeguards one's right to marry the person of

one's choice?

(a) Article 19

(b) Article 21

(c) Article 25

(d) Article 29

Sol. 36 (b) Article 21

Source: PrepMate Current Affairs

Page 36: UPSC 2019 PAPER...UPSC 2019 PAPER SET- D 1. With reference to Asian Infrastructure Investment Bank (AIIB), consider the following statements 1. AIIB has more than 80 member nations

Recently, Supreme Court has in two different cases held that one's right to marry the

person of one's choice comes within the purview of Article 21. These cases relate to (1)

Decision of a hindu girl in Kerala named Hadiya to marry muslim man of his choice and (2)

Matter of honour killing.

37. Consider the following statements:

1. According to the Indian Patents Act, a biological process to create a seed can be patented

in India.

2. In India, there is no Intellectual Property Appellate Board.

3. Plant varieties are not eligible to be patented in India.

Which of the statements given above is/are correct?

(a) 1 and 3 only

(b) 2 and 3 only

Page 37: UPSC 2019 PAPER...UPSC 2019 PAPER SET- D 1. With reference to Asian Infrastructure Investment Bank (AIIB), consider the following statements 1. AIIB has more than 80 member nations

(c) 3 only

(d) 1, 2 and 3

Sol. 37 (c) 3 only

Topic: Economics

All the answer options have Statement 3. Thus, there is no need to evaluate Statement 3.

Statement 2 is incorrect: The Intellectual Property Appellate Board (IPAB) was constituted

in 2003 to hear appeals against the decisions of the registrar under the Indian Trademarks

Act, 1999 and the Geographical Indications of Goods (Registration and Protection) Act,

1999. IPAB is time and again mentioned in current affairs.

Statement 1 is incorrect: Plants and animals in whole or any part thereof including seeds,

varieties and species and biological processes for production or propagation of plants and

animals are not patentable in India.

38. Consider the following statements:

The Environment Protection Act, 1986 empowers the Government of India to

1. state the requirement of public participation in the process of environmental protection,

and the procedure and manner in which it is sought

2. lay down the standards for emission or discharge of environmental pollutants from

various sources

Which of the statements given above is/ are correct?

(a) 1 only

(b) 2 only

(c) Both 1 and 2

(d) Neither 1 nor 2

Sol. 38 (b) 2 only

Topic: Environment

Statement 2 is correct. The Environment Protection Act, 1986 empowers the Government

of India to lay down the standards for emission or discharge of environmental pollutants

from various sources.

Statement 1 is incorrect. There is no provision in the act which empowers the Government

of India to state the requirement of public participation in the process of environmental

protection, and the procedure and manner in which it is sought.

Page 38: UPSC 2019 PAPER...UPSC 2019 PAPER SET- D 1. With reference to Asian Infrastructure Investment Bank (AIIB), consider the following statements 1. AIIB has more than 80 member nations

39. As per the Solid Waste Management Rules, 2016 in India, which one of the following

statements is correct?

(a) Waste generator has to segregate waste into five categories.

(b) The Rules are applicable to notified urban local bodies, notified towns and all industrial

townships only.

(c) The Rules provide for exact and elaborate criteria for the identification of sites for

landfills and waste processing facilities.

(d) It is mandatory on the part of waste generator that the waste generated in one district

cannot be moved to another district.

Sol. 39 (c) The Rules provide for exact and elaborate criteria for the identification of sites

for landfills and waste processing facilities.

Topic: Current Affairs

Solid waste management rules, 2016 have been regularly in news.

Option (a) is incorrect: Waste generators would now have to now segregate waste into

three categories (not five) – Biodegradables, Dry (Plastic, Paper, metal, wood, etc.) and

Domestic Hazardous waste (diapers, napkins, mosquito repellants, cleaning agents etc.)

before handling it over to the collector.

Option (b) is incorrect: The Rules are applicable beyond notified urban local bodies,

notified towns and all industrial townships, and will extend to all other urban

agglomerations such as census towns, notified industrial townships, areas under the

control of Indian Railways,

Option (c): is correct: The landfills shall be 100 meters away from the river; 200 meters

from a pond; 500 meters away from the highway, habitations, public parks and water

supply wells and 20 km away from airports/airbase. The construction of landfills on hills

shall be avoided. Land for construction of sanitary landfills in hilly areas shall be identified

in the plain areas.

Option (d) is incorrect: The bio-degradable waste should be processed, treated and

disposed of within the premises of generator as far as possible. The residual waste shall be

given to the waste collectors or agency as directed by the local authority. There is no such

restriction that it is mandatory on the part of waste generator that the waste generated in

one district cannot be moved to another district.

Page 39: UPSC 2019 PAPER...UPSC 2019 PAPER SET- D 1. With reference to Asian Infrastructure Investment Bank (AIIB), consider the following statements 1. AIIB has more than 80 member nations

40. Consider the following statements:

As per the Industrial Employment (Standing Orders) Central (Amendment) Rules, 2018

1. if rules for fixed-term employment are implemented, it becomes easier for the

firms/companies to layoff workers.

2. no notice of termination of employment shall be necessary in the case of temporary

workman.

Which of the statements given above is / are correct?

(a) 1 only

(b) 2 only

(c) Both 1 and 2

(d) Neither 1 nor 2

Sol. 40 (c) Both 1 and 2

Topic: Current Affairs

This question demands knowledge of Industrial Employment (Standing orders) Central

(Amendment) Rules, 2018. These rules were passed to give flexibility to businesses in

India.

Statement 1 is correct: If rules for fixed team employment are implemented, it become

easier for the firms/companies to lay off workers.

Statement 2 is correct: No notice of termination of employment shall be necessary in the

case of temporary workman. However, there are certain conditions:

Provided that a temporary workman, who has completed three months continuous

service, shall be given two weeks’ notice of the intention to terminate his

employment.

Provided further that when the services of a temporary workman, who has not

completed three month’s continuous service, are terminated before the completion

of the term of employment given to him, he shall be informed of the reasons for

termination in writing.

Page 40: UPSC 2019 PAPER...UPSC 2019 PAPER SET- D 1. With reference to Asian Infrastructure Investment Bank (AIIB), consider the following statements 1. AIIB has more than 80 member nations

41. In the context of digital technologies for entertainment, consider the following

statements:

1. In Augmented Reality (AR), a simulated environment is created and the physical world is

completely shut out.

2. In Virtual Reality (VR), images generated from a computer are projected onto real-life

objects or surroundings.

3. AR allows individuals to be present in the world and improves the experience using the

camera of smart-phone or PC.

4. VR closes the world, and transposes an individual, providing complete immersion

experience.

Which of the statements given above is / are correct?

(a) 1 and 2 only

(b) 3 and 4

(c) 1, 2 and 3

(d) 4 only

Sol. 41 (b) 3 and 4

Topic: Science & Technology

Statement 1 is incorrect and Statement 3 is correct: Augmented reality (AR) adds digital

elements to a live view often by using the camera on a smartphone. Examples of augmented

reality experiences include Snapchat lenses and games such as Pokemon Go.

Statement 2 is incorrect and Statement 4 is correct: Virtual reality (VR) refer to complete

immersion experience that shuts out the physical world. Using VR devices users can be

transported to imagined environments such as among the dinosaurs or within a haunted

house.

42. The word 'Denisovan' is sometimes mentioned in media in reference to

(a) fossils of a kind of dinosaurs

(b) an early human species

(c) a cave system found in North-East India.

(d) a geological period in the history of Indian subcontinent

Sol. 42 (b) an early human species

Page 41: UPSC 2019 PAPER...UPSC 2019 PAPER SET- D 1. With reference to Asian Infrastructure Investment Bank (AIIB), consider the following statements 1. AIIB has more than 80 member nations

Source: Current Affairs

Denisovan is an early human species which was adapted to low-oxygen environment. The

first evidence for Denisovans or Denisova hominins was first discovered in 2008 in a cave

in the Altai mountains in Siberia.

Recently, a fossil jawbone containing molars recovered from Baishiya Karst cave in Xiahe,

Gansu, China shows Denisovans lived in the Tibetan Plateau some 1,60,000 years ago.

43. With reference to the recent developments in science, which one of the following

statements is not correct?

(a) Functional chromosomes can be created by joining segments of DNA taken from cells of

different species.

(b) Pieces of artificial functional DNA can be created in Iaboratories.

(c) A piece of DNA taken out from an animal cell can be made to replicate outside a living

cell in a laboratory.

(d) Cells taken out from plants and animals can be made to undergo cell division in

laboratory petri dishes.

Sol. 43 (a) Functional chromosomes can be created by joining segments of DNA taken from

cells of different species.

Source: PrepMate Science & Technology

This question can be solved through elimination technique.

Let us start from option (d). It is easiest. The statement given in answer option can be

eliminated on the basis of cloning technique. In the process of cloning, cells taken out from

plants and animals undergo cell division in laboratory petri dishes.

Source: PrepMate Science & Technology, Chapter 1, Page 8

Options (b) and (c) can also be eliminated. Synthetic biology is used to create pieces of

artificial functional DNA. It can also be used to replicate DNA outside a living cell.

Source: PrepMate Science & Technology, Chapter 1, Page 7

Page 42: UPSC 2019 PAPER...UPSC 2019 PAPER SET- D 1. With reference to Asian Infrastructure Investment Bank (AIIB), consider the following statements 1. AIIB has more than 80 member nations

44. Consider the following statements:

A digital signature is

1. an electronic record that identifies the certifying authority issuing it

2. used to serve as a proof of identity of an individual to access information or server on

Internet.

3. an electronic method of signing an electronic document and ensuring that the original

content is unchanged

Which of the statements given above is / are correct?

(a) 1 only

(b) 2 and 3 only

(c) 3 only

(d) 1, 2 and 3

Sol. 44 (d) 1, 2 and 3

Source: PrepMate Science & Technology, Chapter 11, Page 159

Statements 1 and 3 are correct.

Page 43: UPSC 2019 PAPER...UPSC 2019 PAPER SET- D 1. With reference to Asian Infrastructure Investment Bank (AIIB), consider the following statements 1. AIIB has more than 80 member nations

Statement 2 is also correct. A digital certificate can also be presented electronically to

prove one’s identity, to access information or services on the Internet.

45. In the context of wearable technology, which of the following tasks is/are accomplished

by wearable devices?

1. Location identification of a person

2. Sleep monitoring of a person

3. Assisting the hearing impaired person

Select the correct answer using the code given below.

(a) 1 only

(b) 2 and 3 only

(c) 3 only

(d) 1, 2 and 3

Sol.45 (d) 1, 2 and 3

Source: PrepMate Current Affairs

Wearable technology is a category of electronic devices that can be worn as accessories,

embedded in clothing, implanted in the user's body, or even tattooed on the skin.

This question can be inferred on the basis of article on Apple watch in Indian express,

which was covered in PrepMate current affairs.

Page 44: UPSC 2019 PAPER...UPSC 2019 PAPER SET- D 1. With reference to Asian Infrastructure Investment Bank (AIIB), consider the following statements 1. AIIB has more than 80 member nations

46. 'RNA interference (RNAi)' technology has gained popularity in the last few years. Why?

1. It is used in developing gene silencing therapies.

2. It can be used in developing therapies for-the treatment of cancer.

3. It can be used to develop hormone replacement therapies.

4. It can be used to produce crop plants that are resistant to viral pathogens.

Select the correct answer using the code given below.

(a) 1, 2 and 4

(b) 2 and 3

(c) 1 and 3

(d) 1 and 4 only

Sol. 46 (a) 1, 2 and 4

Source: PrepMate General Science Book, Biology Section, Chapter 4, Page 233

This question can be solved by understanding functioning of RNA.

Statement 1 is correct. As RNA transcribes genetic information from the DNA found in a

cell’s nucleus and then carries this information to other cell organelles, RNAi is used in

developing gene silencing therapies.

Page 45: UPSC 2019 PAPER...UPSC 2019 PAPER SET- D 1. With reference to Asian Infrastructure Investment Bank (AIIB), consider the following statements 1. AIIB has more than 80 member nations

Statement 2 is correct. Cancer is abnormal growth of body cells which is related to DNA

present in them. As DNA are expressed through RNA, RNAi can be used in developing

therapies for-the treatment of cancer.

Statement 3 is incorrect. So far, genetic information has not been intrinsically related to our

hormone system. Thus, RNAi It cannot be used to develop hormone replacement therapies.

Statement 4 is correct. RNA can impact expression of DNA which may make plant

vulnerable to viral pathogens. Thus, RNAi can be used to produce crop plants that are

resistant to viral pathogens.

47. Recently, scientists observed the merger of giant 'blackholes' billions of light-years

away from the Earth. What is the significance of this observation?

(a) 'Higgs boson particles' were detected.

(b) 'Gravitational waves' were detected.

(c) Possibility of inter-galactic space travel through 'wormhole' was confirmed.

Page 46: UPSC 2019 PAPER...UPSC 2019 PAPER SET- D 1. With reference to Asian Infrastructure Investment Bank (AIIB), consider the following statements 1. AIIB has more than 80 member nations

(d) It enabled the scientists to understand 'singularity'.

Sol. 47 (b) 'Gravitational waves' were detected.

Source: PrepMate Science & Technology Book, Chapter 10, Page 148

48. Which of the following are the reasons for the occurrence of multi-drug resistance in

microbial pathogens in India?

1. Genetic predisposition of some people

2. Taking incorrect doses of antibiotics to cure diseases

3. Using antibiotics in livestock farming

4. Multiple chronic diseases in some people

Select the correct answer using the code given below.

(a) 1 and 2

Page 47: UPSC 2019 PAPER...UPSC 2019 PAPER SET- D 1. With reference to Asian Infrastructure Investment Bank (AIIB), consider the following statements 1. AIIB has more than 80 member nations

(b) 2 and 3 only

(c) 1, 3 and 4

(d) 2, 3 and 4

Sol. 48 (b) 2 and 3 only

Source: PrepMate Science & Technology, Chapter 15, Page 206

Anything which is associated with excessive use of antibiotics is a cause behind antibiotic

resistance.

Statement 2 is correct. Incorrect doses of antibiotics lead to excessive use of antibiotics.

Statement 3 is correct. This statement has been specifically mentioned in current affairs.

Antibiotic resistance in humans has also been traced to resistant microbes originating in

livestock. Thus, increased use of antibiotics in livestock also leads to antibiotic resistance.

Statements 1 and 4 are not directly related to antibiotic resistance.

49. What is Cas9 protein that is often mentioned in news?

(a) A molecular scissors used in targeted gene editing

(b) A biosensor used in the accurate detection of pathogens in patients

Page 48: UPSC 2019 PAPER...UPSC 2019 PAPER SET- D 1. With reference to Asian Infrastructure Investment Bank (AIIB), consider the following statements 1. AIIB has more than 80 member nations

(c) A gene that makes plants pest-resistant

(d) A herbicidal substance synthesized in genetically modified crops

Sol. 49 (a) A molecular scissors used in targeted gene editing

Source: PrepMate Current Affairs

50. Which one of the following statements is not correct?

(a) Hepatitis B virus is transmitted much like HIV.

(b) Hepatitis B, unlike Hepatitis C, does not have a vaccine.

(c) Globally, the number of people infected with Hepatitis B and C viruses are several times

more than those infected with HIV.

Page 49: UPSC 2019 PAPER...UPSC 2019 PAPER SET- D 1. With reference to Asian Infrastructure Investment Bank (AIIB), consider the following statements 1. AIIB has more than 80 member nations

(d) Some of those infected with Hepatitis Band C viruses do not show the symptoms for

many years.

Sol. 50 (b) Hepatitis B, unlike Hepatitis C, does not have a vaccine.

Source: PrepMate Current Affairs

This question has been picked up from a news article in The Hindu. All the answer options

are from single news article. This article was shared in PrepMate News Juice.

51. With reference to Mughal India, what is/are the difference/differences between

Jagirdar and Zamindar?

1. Jagirdars were holders of land assignments in lieu of judicial and police duties, whereas

Zamindars were holders of revenue rights without obligation to perform any duty other

than revenue collection.

Page 50: UPSC 2019 PAPER...UPSC 2019 PAPER SET- D 1. With reference to Asian Infrastructure Investment Bank (AIIB), consider the following statements 1. AIIB has more than 80 member nations

2. Land assignments to Jagirdars were hereditary and revenue rights of Zamindars were

not hereditary.

Select the correct answer using the code given below.

(a) 1 only

(b) 2 only

(c) Both 1 and 2

(d) Neither 1 nor 2

Sol. 51 (d) Neither 1 nor 2

Source: PrepMate Ancient and Medieval History & Culture, Chapter 16, Page 217

Statement 2 is incorrect: Akbar introduced mansabdari system which was based on merit.

The system included all the officials included nobles which were also jagirdars. Thus, we

can conclude that land assignments to jagirdars were not hereditary.

Page 51: UPSC 2019 PAPER...UPSC 2019 PAPER SET- D 1. With reference to Asian Infrastructure Investment Bank (AIIB), consider the following statements 1. AIIB has more than 80 member nations

Statement 1 is also incorrect. Jagirdars included people such as nobles. They were not

required to perform judicial and police duties.

52. With reference to land reforms in independent India, which one of the following

statements is correct?

(a) The ceiling laws were aimed at family holdings and not individual holdings.

(b) The major aim of land reforms was providing agricultural land to all the landless.

(c) It resulted in cultivation of cash crops as a predominant form of cultivation.

(d) Land reforms permitted no exemptions to the ceiling limits.

Sol. 52 (b) The major aim of land reforms was providing agricultural land to all the

landless.

Source: PrepMate Economics, Chapter 12, Page 184

Option (a) is incorrect. Ceiling laws were aimed at both family and individual holdings.

Option (c) is incorrect. Land reforms did not encourage cultivation of cash crops.

Option (d) is incorrect. There were exceptions to land ceiling limits.

Option (b) is the correct answer. The major aim of land reforms was providing agricultural

land to all the landless.

Page 52: UPSC 2019 PAPER...UPSC 2019 PAPER SET- D 1. With reference to Asian Infrastructure Investment Bank (AIIB), consider the following statements 1. AIIB has more than 80 member nations

53. The Global Competitiveness Report is published by the

(a) International Monetary Fund

(b) United Nations Conference on Trade and Development

(c) World Economic Forum

(d) World Bank

Sol. 53(c) World Economic Forum

Source: PrepMate International Organizations and Bilateral Relations, Chapter 7, Page 166

54. Consider the following statements about 'the Charter Act of 1813':

1. It ended the trade monopoly of the East India Company in India except for trade in tea

and trade with China.

2. It asserted the sovereignty of the British Crown over the Indian territories held by the

Company.

Page 53: UPSC 2019 PAPER...UPSC 2019 PAPER SET- D 1. With reference to Asian Infrastructure Investment Bank (AIIB), consider the following statements 1. AIIB has more than 80 member nations

3. The revenues of India were now controlled by the British Parliament.

Which of the statements given above are correct?

(a) 1 and 2 only

(b) 2 and 3 only

(c) 1 and 3 only

(d) 1, 2 and 3

Sol. 54 (a) 1 and 2 only

Source of 1st Statement : PrepMate Modern History Book, Chapter 7, Page 63

Statement 1 is correct: Although the Act renewed the Charter for a further period of twenty

years, it took away the monopoly of the Company to trade in India except trade in tea and

trade with China.

Statement 2 is correct The 1813 Act mentioned about sovereignty of the British Crown

over the Indian territories held by the Company.

Statement 3 is not correct: From 1853 Act, the revenues of India were controlled by the

British Parliament.

55. With reference to Swadeshi Movement, consider the following statements:

1. It contributed to the revival of the indigenous artisan crafts and industries.

2. The National Council of Education was established as a part of Swadeshi Movement.

Which of the statements given above is/are correct?

(a) 1 only

(b) 2 only

(c) Both 1 and 2

(d) Neither 1 nor 2

Sol. 55 (c) Both 1 and 2

Source: PrepMate Modern History Book, Chapter 9, Page 98

Page 54: UPSC 2019 PAPER...UPSC 2019 PAPER SET- D 1. With reference to Asian Infrastructure Investment Bank (AIIB), consider the following statements 1. AIIB has more than 80 member nations

56. Consider the following pairs:

Movement/Organization- Leader

1. All India Anti-Untouchability League - Mahatma Gandhi

2. All India Kisan Sabha - Swami Sahajanand Saraswati

3. Self-Respect Movement - E. V. Ramaswami Naicker

Which of the pairs given above is/are correctly matched?

(a) 1 only

(b) 1 and 2 only

(c) 2 and 3 only

(d) 1, 2 and 3

Sol. 56 (d) 1, 2 and 3

Source: PrepMate Modern History Book; Statement 1 – Chapter 3, Page 26; Statement 2 –

Chapter 4, Page 45; Statement 3 – Chapter 3, Page 28

Statement 1

Page 55: UPSC 2019 PAPER...UPSC 2019 PAPER SET- D 1. With reference to Asian Infrastructure Investment Bank (AIIB), consider the following statements 1. AIIB has more than 80 member nations

Statement 2

Statement 3

57. Which one of the following is not a Harappan site?

(a) Chanhudaro

(b) Kot Diji

(c) Sohgaura

(d) Desalpur

Sol. 57 (c) Sohgaura

Source: PrepMate Ancient and Medieval History & Culture, Chapter 7 answer explanations,

Page 403

Sohgaura is a mauryan period site.

Page 56: UPSC 2019 PAPER...UPSC 2019 PAPER SET- D 1. With reference to Asian Infrastructure Investment Bank (AIIB), consider the following statements 1. AIIB has more than 80 member nations

58. In which of the following relief sculpture inscriptions is 'Ranyo Ashoka' (King Ashoka)

mentioned along with the stone portrait of Ashoka?

(a) Kanganahalli

(b) Sanchi I

(c) Shahbazgarhi

(d) Sohgaura

Sol. (a) Kanganahalli

Topic: Ancient History

The Stone Portrait of Ashoka at Kanaganahalli

Page 57: UPSC 2019 PAPER...UPSC 2019 PAPER SET- D 1. With reference to Asian Infrastructure Investment Bank (AIIB), consider the following statements 1. AIIB has more than 80 member nations

The inscription below the portrait written in brahmi script reads “Ranyo Ashoka” (King

Ashoka).

59. Consider the following:

1. Deification of the Buddha

2. Treading the path of Bodhisattvas

3. Image worship and rituals

Which of the above is/are the feature/ features of Mahayana Buddhism?

(a) 1 only

(b) 1 and 2 only

(c) 2 and 3 only

(d) 1, 2 and 3

Sol. 59 (d) 1, 2 and 3

Source: PrepMate Ancient and Medieval History & Culture, Chapter 5, Page 61

Page 58: UPSC 2019 PAPER...UPSC 2019 PAPER SET- D 1. With reference to Asian Infrastructure Investment Bank (AIIB), consider the following statements 1. AIIB has more than 80 member nations

60. With reference to forced labour (Vishti) in India during the Gupta period, which one of

the following statements is correct?

(a) It was considered a source of income for the State, a sort of tax paid by the people.

(b) It was totally absent in the Madhya Pradesh and Kathiawar regions of the Gupta

Empire.

(c) The forced labourer was entitled to weekly wages.

(d) The eldest son of the labourer was sent as the forced labourer.

Sol. 60 (a) It was considered a source of income for the State, a sort of tax paid by the

people.

Topic: Ancient History

During Gupta period, Vishti was considered a source of income for the State, a sort of tax

paid by the people.

61. Which one of the following groups of plants was domesticated in the 'New World' and

introduced into the 'Old World'?

(a) Tobacco, cocoa and rubber

(b)Tobacco, cotton and rubber.

(c) Cotton, coffee and sugarcane

Page 59: UPSC 2019 PAPER...UPSC 2019 PAPER SET- D 1. With reference to Asian Infrastructure Investment Bank (AIIB), consider the following statements 1. AIIB has more than 80 member nations

(d) Rubber, coffee and wheat

Sol. 61 (a) Tobacco, cocoa and rubber

Topic: World History

This question is totally unexpected. Old World refers to Africa, Asia, and Europe while New

World refers to the Americas, including North America, Central America, and South

America. Europe, Asia, and Africa shared a common agricultural history. The Old World

crops include wheat, rye, oats, lentils, and barley. Such crops did not exist in the Americas

until their introduction in the 1490s by post-Columbian contact. The famous New World

crops include rubber, tobacco, sunflower, cocoa, and cashew. The merchants of Europe

took products of the New World to Europe, including tobacco, potatoes, cocoa and rubber.

62. Consider the following statements:

1. Asiatic lion is naturally found in India only.

2. Double-humped camel is naturally found in India only.

3. One-horned rhinoceros is naturally found in India only.

Which of the statements given above is / are correct?

(a) 1 only

(b) 2 only

(c) 1 and 3 only

(d) 1, 2 and 3

Sol. 62 (a) 1 only

Source: PrepMate Environment & Biodiversity book, Chapter 14, Page 233 and PrepMate

Current affairs

Statement 1 is correct.

Page 60: UPSC 2019 PAPER...UPSC 2019 PAPER SET- D 1. With reference to Asian Infrastructure Investment Bank (AIIB), consider the following statements 1. AIIB has more than 80 member nations

Statement 3 is incorrect. One-horned rhinoceros is not found only in India.

63. Consider the following pairs:

Famous place- River

1. Pandharpur- Chandrabhaga

2. Tiruchirappalli- Cauvery

3. Hampi- Malaprabha

Which of the pairs given above are correctly matched?

(a) 1 and 2 only

(b) 2 and 3 only

(c) 1 and 3 only

(d) 1, 2 and 3

Page 61: UPSC 2019 PAPER...UPSC 2019 PAPER SET- D 1. With reference to Asian Infrastructure Investment Bank (AIIB), consider the following statements 1. AIIB has more than 80 member nations

Sol. 63 (a) 1 and 2 only

Source: PrepMate Geography, Chapter 19, Page 239

This is an interesting question. It can be solved by ascertaining validity of only Statement 3.

Statement 3 is incorrect. Malaprabha rises in the Western Ghats and flows in North-West

Karnataka, before joining river Krishna. On the other hand, Hampi is in eastern part of

Karnataka.

64. In a given year in India, official poverty lines are higher in some States than in others

because

(a) poverty rates vary from State to State

(b) price levels vary from State to State

(c) Gross State Product varies from State to State

(d) quality of public distribution varies from State to State

Sol. 64 (b) price levels vary from State to State

Source: PrepMate Economics Book, Chapter 1, Page 13

This question requires understanding of concept of poverty line. Poverty line is based on

the ability to meet expenditure to fulfill bare minimum needs. This expenditure varies from

Page 62: UPSC 2019 PAPER...UPSC 2019 PAPER SET- D 1. With reference to Asian Infrastructure Investment Bank (AIIB), consider the following statements 1. AIIB has more than 80 member nations

state to state because price levels vary from state to state. Price levels are taken into

consider through Consumer Price Index.

65. In the context of which of the following do some scientists suggest the use of cirrus

cloud thinning technique and the injection of sulphate aerosol into stratosphere?

(a) Creating the artificial rains in some regions

(b) Reducing the frequency and intensity of tropical cyclones

(c) Reducing the adverse effects of solar wind on the Earth

(d) Reducing the global warming

Sol. 65 (d) Reducing the global warming

Source: PrepMate Current Affairs

Page 63: UPSC 2019 PAPER...UPSC 2019 PAPER SET- D 1. With reference to Asian Infrastructure Investment Bank (AIIB), consider the following statements 1. AIIB has more than 80 member nations

This question is asked from an article in Indian Express. The same article was shared in

PrepMate news juice.

66. In the context of which one of the following are the terms 'pyrolysis and plasma

gasification' mentioned?

(a) Extraction of rare earth elements

(b) Natural gas extraction technologies

(c) Hydrogen fuel-based automobiles

(d) Waste-to-energy technologies

Sol. 66 (d) Waste-to-energy technologies

Source: PrepMate Current Affairs

This question is asked from an article in PIB. The same article was shared in PrepMate

news juice.

Page 64: UPSC 2019 PAPER...UPSC 2019 PAPER SET- D 1. With reference to Asian Infrastructure Investment Bank (AIIB), consider the following statements 1. AIIB has more than 80 member nations

67. Which of the following are in Agasthyamala Biosphere Reserve?

(a) Neyyar, Peppara and Shendurney Wildlife Sanctuaries; and Kalakad Mundanthurai

Tiger Reserve

(b) Mudumalai, Sathyamangalam and Wayanad Wildlife Sanctuaries; and Silent Valley

National Park

(c) Kaundinya, Gundla Brahme-swaram and Papikonda Wildlife Sanctuaries; and Mukurthi

National Park

(d) Kawal and Sri Venkateswara Wildlife Sanctuaries; and Nagarjunasagar-Srisailam Tiger

Reserve

Page 65: UPSC 2019 PAPER...UPSC 2019 PAPER SET- D 1. With reference to Asian Infrastructure Investment Bank (AIIB), consider the following statements 1. AIIB has more than 80 member nations

Sol. 67 (a) Neyyar, Peppara and Shendurney Wildlife Sanctuaries; and Kalakad

Mundanthurai Tiger Reserve

Source: PrepMate Environment & Biodiversity Book, Chapter 13, Page 212

This question can also be attempted using elimination technique. Candidate can compare

the location of given national parks, wildlife sanctuaries and tiger reserves with that of the

Agasthyamala biosphere reserve and eliminate wrong answer choices. It is important to

note a candidate can learn location of only important national parks, wildlife sanctuaries

and tiger reserves. Learning location of all the national parks, wildlife sanctuaries and tiger

reserves is cumbersome and is not recommended.

68. Consider the following statements:

1. Some species of turtles are herbivores.

2. Some species of fish are herbivores.

3. Some species of marine mammals are herbivores.

Page 66: UPSC 2019 PAPER...UPSC 2019 PAPER SET- D 1. With reference to Asian Infrastructure Investment Bank (AIIB), consider the following statements 1. AIIB has more than 80 member nations

4. Some species of snakes are viviparous.

Which of the statements given above are correct?

(a) 1 and 3 only

(b) 2, 3 and 4 only

(c) 2 and 4 only

(d) 1, 2, 3 and 4

Sol. 68 (d) 1, 2, 3 and 4

Source of 3rd statement: PrepMate Environment & Biodiversity book, Chapter 12, Page 200

Statement 3 is correct. Dugong is a marine mammal. It survives on sea grass. Thus, we can

conclude that some species of marine mammals are herbivores.

Statement 2 is correct. It is a common fact that some species of fish are herbivores.

Statement 1 is correct. There are some turtle species such as green turtles which are

herbivores.

We can answer the question by knowing the validity of just these three statements.

69. Consider the following pairs:

Wildlife: Naturally found in

1. Blue-finned Mahseer: Cauvery River

2. Irrawaddy Dolphin: Chambal River

Page 67: UPSC 2019 PAPER...UPSC 2019 PAPER SET- D 1. With reference to Asian Infrastructure Investment Bank (AIIB), consider the following statements 1. AIIB has more than 80 member nations

3. Rusty-spotted Cat: Eastern Ghats

Which of the pairs given above are correctly matched?

(a) 1 and 2 only

(b) 2 and 3 only

(c) 1 and 3 only

(d) 1, 2 and 3

Sol. 69 (c) 1 and 3 only

Source: PrepMate Environment & Biodiversity Book, Chapter 14, Page 235

We can solve this question by knowing the validity of statement 2 only.

Statement 2 is incorrect. Freshwater dolphins are present in Ganga and its tributaries, not

Chambal.

Once we rule out statement 2, answer is option (c).

70. Why is there a great concern about the 'microbeads' that are released into

environment?

(a) They are considered harmful to marine ecosystems.

(b) They are considered to cause skin cancer in children.

(c) They are small enough to be absorbed by crop plants in irrigated fields.

(d) They are often found to be used as food adulterants.

Sol. 70 (a) They are considered harmful to marine ecosystems.

Source: PrepMate Current Affairs

Many times there have been articles in newspapers on Microbeads. Here is a one article in

The Hindu, which was shared in PrepMate Current Affairs.

Page 68: UPSC 2019 PAPER...UPSC 2019 PAPER SET- D 1. With reference to Asian Infrastructure Investment Bank (AIIB), consider the following statements 1. AIIB has more than 80 member nations

71. Building 'Kalyaana Mandapas' was a notable feature in the temple construction in the

kingdom of

(a) Chalukya

(b) Chandela

(c) Rashtrakuta

(d) Vijayanagara

Sol. 71 (d) Vijayanagara

Source: PrepMate Ancient and Medieval History & Culture, Chapter 20, Page 289

This question can be attempted from both selection as well as elimination technique.

Option (a) can be eliminated because Chalukyas were associated with cave temples.

Option (b) can be eliminated because Chandela did not emphasise on large Mandapas in

temple. Their famous creation is Kandariya Mahadeva temple in Khajurao.

Option (c) can be eliminated because Rashtrakutas were also not associated with large

Mandapas.

Page 69: UPSC 2019 PAPER...UPSC 2019 PAPER SET- D 1. With reference to Asian Infrastructure Investment Bank (AIIB), consider the following statements 1. AIIB has more than 80 member nations

Option (d) is the correct answer. 'Kalyaana Mandapas' or wedding hall was built in temples

by kingdom of Vijayanagara.

72. Consider the following statements:

Page 70: UPSC 2019 PAPER...UPSC 2019 PAPER SET- D 1. With reference to Asian Infrastructure Investment Bank (AIIB), consider the following statements 1. AIIB has more than 80 member nations

1. In the revenue administration of Delhi Sultanate, the in-charge of revenue collection was

known as 'Amil'.

2. The Iqta system of Sultans of Delhi was an ancient indigenous institution.

3. The office of 'Mir Bakshi' came into existence during the reign of Khalji Sultans of Delhi.

Which of the statements given above is/are correct?

(a) 1 only

(b) 1 and 2 only

(c) 3 only

(d) 1, 2 and 3

Sol. 72 (a) 1 only

Source: PrepMate Ancient and Medieval History & Culture, Chapter 13, Page 181 and

Chapter 16, Page 217

Statement 2 is incorrect.

Statement 3 is also incorrect. The office of Mir Bakshi came into existence during reign of

Mughals.

Page 71: UPSC 2019 PAPER...UPSC 2019 PAPER SET- D 1. With reference to Asian Infrastructure Investment Bank (AIIB), consider the following statements 1. AIIB has more than 80 member nations

73. Consider the following statements:

1. Saint Nimbarka was a contemporary of Akbar.

2. Saint Kabir was greatly influenced by Shaikh Ahmad Sirhindi.

Which of the statements given above is/are correct?

(a) 1 only

(b) 2 only

(c) Both 1 and 2

(d) Neither 1 nor 2

Sol. 73 (d) Neither 1 nor 2

Source: PrepMate Ancient and Medieval History & Culture, Chapter 14, Page 190 and 192

Statement 1 is incorrect. Saint Nimbarka (11th century) was not contemporary of Akbar

(1542-1605).

Statement 2 is incorrect. Saint kabir lived earlier than Shaikh Ahmad Sirhindi.

Page 72: UPSC 2019 PAPER...UPSC 2019 PAPER SET- D 1. With reference to Asian Infrastructure Investment Bank (AIIB), consider the following statements 1. AIIB has more than 80 member nations

74. With reference to the British colonial rule in India, consider the following statements:

1. Mahatma Gandhi was instrumental in the abolition of the system of 'indentured labour'.

2. In Lord Chelmsford's 'War Conference', Mahatma Gandhi did not support the resolution

on recruiting Indians for World War.

3. Consequent upon the breaking of Salt Law by Indian people, the Indian National

Congress was declared illegal by the colonial rulers.

Which of the statements given above are correct?

(a) 1 and 2 only

(b) 1 and 3 only

(c) 2 and 3 only

(d) 1, 2 and 3

Sol. 74(b) 1 and 3 only

Source of 2nd and 3rd statement: PrepMate Modern History Book, Chapter 9, Page 102 and

Chapter 14, Page 174

Statement 1 is correct. Mahatma Gandhi pressurized authorities to abolish indentured

labour in India. Due to this pressure, indentured labour was abolished in the year 1917.

Page 73: UPSC 2019 PAPER...UPSC 2019 PAPER SET- D 1. With reference to Asian Infrastructure Investment Bank (AIIB), consider the following statements 1. AIIB has more than 80 member nations

Statement 2 is incorrect. Mahatma Gandhi supported British World war I efforts. Rather, he

was conferred with Kaiser-e-hind to support british war efforts.

Chapter 9, Page 102

Statement 3 is correct.

Chapter 14, Page 174

75. With reference to Indian National Movement, consider the following pairs:

Person- Position held

1. Sir Tej Bahadur Sapru- President, All India Liberal Federation

2. K. C. Neogy- Member, The Constituent Assembly

3. P. C. Joshi- General Secretary, Communist Party of India

Page 74: UPSC 2019 PAPER...UPSC 2019 PAPER SET- D 1. With reference to Asian Infrastructure Investment Bank (AIIB), consider the following statements 1. AIIB has more than 80 member nations

Which of the pairs given above is/are correctly matched?

(a) 1 only

(b) 1 and 2 only

(c) 3 only

(d) 1, 2 and 3

Sol. 75 (d) 1, 2 and 3

Source of 1st Statement: PrepMate Modern History Book, Chapter 15, Page 202

Statement 1 is correct.

Statement 3 is correct. P. C. Joshi was General Secretary, Communist Party of India.

Once we know that Statements 1 and 3 are correct, answer is option (d).

76. With reference to Mian Tansen, which one of the following statements is not correct?

(a) Tansen was the title given to him by Emperor Akbar.

(b) Tansen composed Dhrupads on Hindu gods and goddesses.

Page 75: UPSC 2019 PAPER...UPSC 2019 PAPER SET- D 1. With reference to Asian Infrastructure Investment Bank (AIIB), consider the following statements 1. AIIB has more than 80 member nations

(c) Tansen composed songs on his patrons.

(d) Tansen invented many Ragas.

Sol. 76 (a) Tansen was the title given to him by Emperor Akbar.

Topic: Medieval History

Option (a) is the answer. The actual name of Tansen was Ram Tanu Pandey. He was given

title of ‘Tansen’ by Raja Man Singh Tomar of Gwalior. Emperor Akbar gave him title of ‘Mian’.

77. Who among the following Mughal Emperors shifted emphasis from illustrated

manuscripts to album and individual portrait?

(a) Humayun

(b) Akbar

(c) Jahangir

(d) Shah Jahan

Sol. 77 (c) Jahangir

Source: PrepMate Ancient and Medieval History & Culture, Chapter 21, Page 321

The screenshot below taken from PrepMate Ancient and Medieval History & Culture,

Chapter 21, Page 320 prove that illustrated manuscripts were popular during the reign of

Akbar.

Page 76: UPSC 2019 PAPER...UPSC 2019 PAPER SET- D 1. With reference to Asian Infrastructure Investment Bank (AIIB), consider the following statements 1. AIIB has more than 80 member nations

78. Which one of the following National Parks lies completely in the temperate alpine zone?

(a) Manas National Park

(b) Namdapha National Park

(c) Neora Valley National Park

(d) Valley of Flowers National Park

Sol. 78 (d) Valley of Flowers National Park

Source: PrepMate Geography, Chapter 25, Page 314

The question stem asks about national park which lies completely in the temperate alpine

zone. Thus, the national park should be completely based in higher Himalayas.

Page 77: UPSC 2019 PAPER...UPSC 2019 PAPER SET- D 1. With reference to Asian Infrastructure Investment Bank (AIIB), consider the following statements 1. AIIB has more than 80 member nations

79. Atal Innovation Mission is set up under the

(a) Department of Science and Technology

(b) Ministry of Employment

(c) NITI Aayog

(d) Ministry of Skill Development and Entrepreneurship

Sol. 79 (c) NITI Aayog

Source: PrepMate Current Affairs

Page 78: UPSC 2019 PAPER...UPSC 2019 PAPER SET- D 1. With reference to Asian Infrastructure Investment Bank (AIIB), consider the following statements 1. AIIB has more than 80 member nations

80. On 21st June, the Sun

(a) does not set below the horizon at the Arctic Circle

(b) does not set below the horizon at Antarctic Circle

(c) shines vertically overhead at noon on the Equator

(d) shines vertically overhead at the Tropic of Capricorn

Sol. 80 (a) does not set below the horizon at the Arctic Circle

Source: PrepMate Geography, Chapter 3, Page 24

On 21st June, Sun is vertically overhead at the Tropic of Cancer and there is six months of

daylight on north pole. Thus, On 21st June, the Sun does not set below the horizon at the

Arctic Circle.

Page 79: UPSC 2019 PAPER...UPSC 2019 PAPER SET- D 1. With reference to Asian Infrastructure Investment Bank (AIIB), consider the following statements 1. AIIB has more than 80 member nations

81. Consider the following statements:

1. Agricultural soils release nitrogen oxides into environment.

2. Cattle release ammonia into environment.

Page 80: UPSC 2019 PAPER...UPSC 2019 PAPER SET- D 1. With reference to Asian Infrastructure Investment Bank (AIIB), consider the following statements 1. AIIB has more than 80 member nations

3. Poultry industry releases reactive nitrogen compounds into environment.

Which of the statements given above is/are correct?

(a) 1 and 3 only

(b) 2 and 3 only

(c) 2 only

(d) 1, 2 and 3

Sol. 81 (b) 2 and 3

Source: PrepMate Environment & Biodiversity, Chapter 2, Page 24 and Current Affairs

Statement 1 is incorrect. Agricultural soils release free atmospheric nitrogen (and not

nitrogen oxides) into environment.

Statement 2 is correct. Ammonia is released into the environment on excretion and death

of cattle.

Statement 3 is correct. Poultry industry releases reactive nitrogen compounds into

environment.

It is interesting to note that the Statements of this question were directly picked from an

article published in The Hindu.

Page 81: UPSC 2019 PAPER...UPSC 2019 PAPER SET- D 1. With reference to Asian Infrastructure Investment Bank (AIIB), consider the following statements 1. AIIB has more than 80 member nations

82. What is common to the places known as Aliyar, Isapur and Kangsabati?

(a) Recently discovered uranium deposits

(b) Tropical rain forests

(c) Underground cave systems

(d) Water reservoirs

Sol. 82 (d) Water reservoirs

Page 82: UPSC 2019 PAPER...UPSC 2019 PAPER SET- D 1. With reference to Asian Infrastructure Investment Bank (AIIB), consider the following statements 1. AIIB has more than 80 member nations

Topic: Current Affairs

83. In the context of proposals to the use of hydrogen-enriched CNG (H-CNG) as fuel for

buses in public transport, consider the following statements:

1. The main advantage of the use of H-CNG is the elimination of carbon monoxide

emissions.

2. H-CNG as fuel reduces carbon dioxide and hydrocarbon emissions.

3. Hydrogen up to one-fifth by volume can be blended with CNG as fuel for buses.

4. H-CNG makes the fuel less expensive than CNG.

Which of the statements given above is / are correct?

(a) 1 only

(b) 2 and 3 only

(c) 4 only

(d) 1, 2, 3 and 4

Sol. 83 (b) 2 and 3 only

Source: PrepMate Current Affairs

This question is directly picked up from an article in Indian Express. This article was

shared in PrepMate News Juice.

Statement 1 is incorrect. H-CNG reduce carbon monoxide emissions by upto 70 %, but does

not eliminate them.

Statement 2 is correct: When compared to other fuels, H-CNG as fuel reduces carbon

dioxide and hydrocarbon emissions.

Statement 3 is correct: The appropriate Hydrogen concentration, which is 18-20% (up to

one-fifth by volume) is blended with CNG.

Statement 4 is incorrect: Presently, H-CNG is costlier than CNG.

Page 83: UPSC 2019 PAPER...UPSC 2019 PAPER SET- D 1. With reference to Asian Infrastructure Investment Bank (AIIB), consider the following statements 1. AIIB has more than 80 member nations

84. Why are dewdrops not formed on a cloudy night?

(a) Clouds absorb the radiation released from the Earth's surface.

(b) Clouds reflect back the Earth's radiation.

(c) The Earth's surface would have low temperature on cloudy nights.

(d) Clouds deflect the blowing wind to ground level.

Sol. 84 (b) Clouds reflect back the Earth's radiation.

Source: PrepMate Geography, Chapter 12, Page 113

This question requires precise application of theoretical concepts. We understand that dew

are tiny droplets of water which develop on surfaces which radiate heat and experience

condensation. The faster the ability to radiate heat, the higher will be the rate of

condensation and dew formation.

Page 84: UPSC 2019 PAPER...UPSC 2019 PAPER SET- D 1. With reference to Asian Infrastructure Investment Bank (AIIB), consider the following statements 1. AIIB has more than 80 member nations

Thus, any factor which increases the rate of heat radiation, will aid in formation of dew

drops. On the other hand, any factor which reduces the rate of heat radiation, will prevent

formation of dew drops.

Option (b) is the correct answer because when clouds reflect back the Earth's radiation,

more heat accumulates at the surface of earth. Accumulation of more heat at surface of

earth prevents condensation and thus, dew formation.

85. Consider the following statements:

1. The 44th Amendment to the Constitution of India introduced an Article placing the

election of the Prime Minister beyond judicial review.

2. The Supreme Court of India struck down the 99th Amendment to the Constitution of

India as being violative of the independence of judiciary.

Which of the statements given above is/are correct?

(a) 1 only

(b) 2 only

(c) Both 1 and 2

(d) Neither 1 nor 2

Sol. 85 (b) 2 only

Source: PrepMate Polity, Chapter 25, Page 283 and Chapter 8, Page 150

Statement 1 is incorrect. PrepMate Polity, Chapter 25, Page 283

Page 85: UPSC 2019 PAPER...UPSC 2019 PAPER SET- D 1. With reference to Asian Infrastructure Investment Bank (AIIB), consider the following statements 1. AIIB has more than 80 member nations

Statement 2 is correct. PrepMate Polity, Chapter 8, Page 150

86. Consider the following statements:

1. The motion to impeach a Judge of the Supreme Court of India cannot be rejected by the

Speaker of the Lok Sabha as per the Judges (Inquiry) Act, 1968.

2. The Constitution of India defines and gives details of what Constitutes 'incapacity and

proved misbehaviour' of the Judges of the Supreme Court of India.

3. The details of the process of impeachment of the Judges of the Supreme Court of India

are given in the Judges (Inquiry) Act, 1968.

4. If the motion for the impeachment of a Judge is taken up for voting, the law requires the

motion to be backed by each House of the Parliament and supported by a majority of total

membership of that House and by not less than two-thirds of total members of that House

present and voting.

Which of the statements given above is/are correct?

(a) 1 and 2

(b) 3 only

(c) 3 and 4 only

(d) 1, 3 and 4

Sol. 86 (c) 3 and 4 only

Page 86: UPSC 2019 PAPER...UPSC 2019 PAPER SET- D 1. With reference to Asian Infrastructure Investment Bank (AIIB), consider the following statements 1. AIIB has more than 80 member nations

Source for 2nd and 4th Statement: PrepMate Polity, Chapter 8, Page 151

Statement 2 is incorrect. The Constitution of India does not define what constitutes

incapacity and proved misbehavior of the Judges of Supreme Court. It is for the Parliament

to decide what constitutes incapacity and proved misbehavior.

Statement 4 is correct. If the motion for the impeachment of a Judge is taken up for voting,

the law requires the motion to be backed by each House of the Parliament and supported

by a majority of total membership of that House and by not less than two-thirds of total

members of that House present and voting.

Statement 3 is correct. The details of the process of impeachment of the Judges of the

Supreme Court of India are given in the Judges (Inquiry) Act, 1968.

Statement 1 is incorrect. The motion to impeach a Judge of the Supreme Court of India can

be rejected by the Speaker of the Lok Sabha as per the Judges (Inquiry) Act, 1968.

87. The Ninth Schedule was introduced in the Constitution of India during the prime

ministership of

(a) Jawaharlal Nehru

(b) Lal Bahadur Shastri

(c) Indira Gandhi

(d) Morarji Desai

Sol.87 (a) Jawaharlal Nehru

Source PrepMate Polity, Chapter 5, Page 44

Page 87: UPSC 2019 PAPER...UPSC 2019 PAPER SET- D 1. With reference to Asian Infrastructure Investment Bank (AIIB), consider the following statements 1. AIIB has more than 80 member nations

88. Consider the following statements:

1. Coal sector was nationalized by the Government of India under Indira Gandhi.

2. Now, coal blocks are allocated on lottery basis.

3. Till recently, India imported coal to meet the shortages of domestic supply, but now India

is self-sufficient in coal production.

Which of the statements given above is/are correct?

(a) 1 only

(b) 2 and 3 only

(c) 3 only

(d) 1, 2 and 3

Sol. 88 (a) 1 only

Source: PrepMate Current Affairs

Statement 3 is incorrect. India imports coal for its consumption.

Statement 1 is correct.

Page 88: UPSC 2019 PAPER...UPSC 2019 PAPER SET- D 1. With reference to Asian Infrastructure Investment Bank (AIIB), consider the following statements 1. AIIB has more than 80 member nations

Statement 2 is incorrect. Now, Coal blocks are allocated on auction basis (and not lottery

basis).

89. Consider the following statements:

1. The Parliament (Prevention of Disqualification) Act, 1959 exempts several posts from

disqualification on the grounds of 'Office of Profit'.

2. The above-mentioned Act was amended five times.

3. The term 'Office of Profit' is well-defined in the Constitution of India.

Which of the statements given above is/are correct?

Page 89: UPSC 2019 PAPER...UPSC 2019 PAPER SET- D 1. With reference to Asian Infrastructure Investment Bank (AIIB), consider the following statements 1. AIIB has more than 80 member nations

(a) 1 and 2 only

(b) 3 only

(c) 2 and 3 only

(d) 1, 2 and 3

Sol. 89 (a) 1 and 2 only

Source: PrepMate Polity, Chapter 8, Page 74

This question can be solved on the basis of Statement 3 only.

Statement 3 is incorrect. The term 'Office of Profit' is not defined in the Constitution of

India. Supreme Court has defined this term.

90. Under which Schedule of the Constitution of India can the transfer of tribal land to

private parties for mining be declared null and void?

(a) Third Schedule

(b) Fifth Schedule

(c) Ninth Schedule

(d) Twelfth Schedule

Sol. 90 (b) Fifth Schedule

Source: PrepMate Polity, Chapter 28, Page 300

The fifth schedule grants powers to Governor of the state to ensure that the administration

of Scheduled areas is in the best interest of tribals.

Page 90: UPSC 2019 PAPER...UPSC 2019 PAPER SET- D 1. With reference to Asian Infrastructure Investment Bank (AIIB), consider the following statements 1. AIIB has more than 80 member nations

91. Recently, there was a growing awareness in our country about the importance of

Himalayan nettle (Girardinia diversifolia) because it is found to be a sustainable source of

(a) anti-malarial drug

(b) biodiesel

(c) pulp for paper industry

(d) textile fibre

Sol. 91 (d) textile fibre

Topic: Current Affairs

Girardinia diversifolia is a fibre yielding plant. It is considered as a sustainable source for

textile fibre.

92. For the measurement/estimation of which of the following are satellite images/remote

sensing data used?

1. Chlorophyll content in the vegetation of a specific location

2. Greenhouse gas emissions from rice paddies of a specific location

3. Land surface temperatures of a specific location

Select the correct answer using the code given below.

(a) 1 only

(b) 2 and 3 only

Page 91: UPSC 2019 PAPER...UPSC 2019 PAPER SET- D 1. With reference to Asian Infrastructure Investment Bank (AIIB), consider the following statements 1. AIIB has more than 80 member nations

(c) 3 only

(d) 1, 2 and 3

Sol. 92 (d) 1, 2 and 3

Source: PrepMate Science & Technology Book, Chapter 3, Page 49

This question requires understanding of capability of remote sensing satellites (RSS). All

the given statements in this question are correct.

Statement 1 is correct. Chlorophyll content in the vegetation of a specific location can be

estimated by observing composition of plants.

Statement 2 is correct. Greenhouse gas emissions are tracked by RSS.

Statement 3 is correct. RSS can track temperature through measurement of radiations.

Page 92: UPSC 2019 PAPER...UPSC 2019 PAPER SET- D 1. With reference to Asian Infrastructure Investment Bank (AIIB), consider the following statements 1. AIIB has more than 80 member nations

93. Consider the following States:

1. Chhattisgarh

2. Madhya Pradesh

3. Maharashtra

4. Odisha

With reference to the States mentioned above, in terms of percentage of forest cover to the

total area of State, which one of the following is the correct ascending order?

(a) 2-3-1-4

(b) 2-3-4-1

(c) 3-2-4-1

(d) 3-2-1-4

Sol. 93 (c) 3-2-4-1

Source: PrepMate Environment & Biodiversity, Chapter 3, Page 38

This question has been asked from State of forest report, 2017. The report is included in

PrepMate Environment & Biodiversity Book.

In the given states, only Chattisgarh falls in category of 33% to 75% area under forest

cover. Thus, it has the maximum percentage of forest cover to the total area.

In the given states, Maharashtra has least percentage of forest cover to the total area. The

eastern part of Maharashtra has lower rainfall and thus it is in nature of savanna. The

western part of Maharashtra is highly urbanized.

94. Which of the following statements are correct about the deposits of 'methane hydrate’?

1. Global warming might trigger the release of methane gas from these deposits.

2. Large deposits of 'methane hydrate' are found in Arctic Tundra and under the seafloor.

3. Methane in atmosphere oxidizes to carbon dioxide after a decade or two.

Select the correct answer using the code given below.

Page 93: UPSC 2019 PAPER...UPSC 2019 PAPER SET- D 1. With reference to Asian Infrastructure Investment Bank (AIIB), consider the following statements 1. AIIB has more than 80 member nations

(a) 1 and 2 only

(b) 2 and 3 only

(c) 1 and 3 only

(d) 1, 2 and 3

Sol. 94 (d) 1, 2 and 3

Source of 1st and 2nd Statement: PrepMate Environment & Biodiversity Book, Chapter 18,

Page 267

Statements 1 and 2 are correct.

Statement 3 is also correct. When methane is anthropogenically emitted, methane is

oxidized in the atmosphere a decade or two later. Once oxidized, the carbon in each

methane molecule is converted to CO2, which then stays in the atmosphere as CO2 for

another century or more.

95. Consider the following:

1. Carbon monoxide

2. Methane

3. Ozone

4. Sulphur dioxide

Which of the above are released into atmosphere due to the burning of crop/biomass

residue?

(a) 1 and 2 only

(b) 2, 3 and 4 only

(c) 1 and 4 only

(d) 1, 2, 3 and 4

Page 94: UPSC 2019 PAPER...UPSC 2019 PAPER SET- D 1. With reference to Asian Infrastructure Investment Bank (AIIB), consider the following statements 1. AIIB has more than 80 member nations

Sol. 95 (d) 1, 2, 3 and 4

Source: PrepMate Current Affairs

Burning of crop /biomass residues generate greenhouse gases (GHGs) such as carbon

dioxide (CO2), carbon monoxide (CO), methane (CH4) and nitrous oxide (N2O) as well as

volatile organic compounds (VOCs), ammonia (NH3), sulphur dioxide (SO2), nitrogen

oxides (NOx) and particulate matter (PM).

Ozone is a secondary pollutant. It does not emerges directly from burning of crop/biomass

residue. However, agricultural residues burning also releases a huge amount of pollutants

to the atmosphere, which apart from the above includes aerosols and hydrocarbons. Haze

formation is one of the major impacts of open burning of rice fields. This haze has become

an important source of tropospheric ozone and the aerosols resulting from such events

have contributed largely to the formation of atmospheric brown clouds.

96. Consider the following pairs:

Sea: Bordering country

1. Adriatic Sea: Albania

2. Black Sea: Croatia

3. Caspian Sea: Kazakhstan

4. Mediterranean Sea: Morocco

Page 95: UPSC 2019 PAPER...UPSC 2019 PAPER SET- D 1. With reference to Asian Infrastructure Investment Bank (AIIB), consider the following statements 1. AIIB has more than 80 member nations

5. Red Sea: Syria

Which of the pair given above are correctly matched?

(a) 1, 2 and 4 only

(b) 1, 3 and 4 only

(c) 2 and 5 only

(d) 1, 2, 3, 4 and 5

Sol. 96 (b) 1, 3 and 4 only

Source: PrepMate Geography

Pair 1 (Adriatic Sea: Albania) is correctly matched. Source: PrepMate Geography, Chapter

16, Page 178 and 181

Pair 2 (Black Sea: Croatia) is not correctly matched. Source: PrepMate Geography, Chapter

16, Page 178

Pair 3 (Caspian Sea: Kazakhstan) is correctly matched. Source: PrepMate Geography,

Chapter 16, Page 187

Page 96: UPSC 2019 PAPER...UPSC 2019 PAPER SET- D 1. With reference to Asian Infrastructure Investment Bank (AIIB), consider the following statements 1. AIIB has more than 80 member nations

Pair 4 (Mediterranean Sea: Morocco) is correctly matched. Source: PrepMate Geography,

Chapter 16, Page 178

Morocco is situated in North Africa.

Pair 5 (Red Sea: Syria) is incorrectly matched. Source: PrepMate Geography, Chapter 16,

Page 183

97. Among the following, which one is the largest exporter of rice in the world in the last

five years?

(a) China

(b) India

(c) Myanmar

(d) Vietnam

Sol. 97 (b) India

Source: PrepMate Geography, Chapter 31, Page 417

98. Consider the following pairs:

Glacier: River

1. Bandarpunch: Yamuna

2. Bara Shigri: Chenab

3. Milam: Mandakini

4. Siachen: Nubra

5. Zemu: Manas

Page 97: UPSC 2019 PAPER...UPSC 2019 PAPER SET- D 1. With reference to Asian Infrastructure Investment Bank (AIIB), consider the following statements 1. AIIB has more than 80 member nations

Which of the pairs given above are correctly matched?

(a) 1, 2 and 4

(b) 1, 3 and 4

(c) 2 and 5

(d) 3 and 5

Sol. 98 (a) 1, 2 and 4

Topic: Geography

Origin Glacier: River

1. Bandarpunch: Yamuna

2. Bara Shigri: Chenab

3. Milam: Gori Ganga

4. Siachen: Nubra

5. Zemu: Teesta

Pairs 1, 2 and 4 are correctly matched.

99. In India, the use of carbofuran, methyl parathion, phorate and triazophos is viewed with

apprehension. These chemicals are used as

(a) pesticides in agriculture

(b) preservatives in processed foods

(c) fruit-ripening agents

(d) moisturising agents in cosmetics

Sol. 99 (a) pesticides in agriculture

Topic: Current Affairs

100. Consider the following statements:

1. Under Ramsar Convention, it is mandatory on the part of the Government of India to

protect and conserve all the wetlands in the territory of India.

2. The Wetlands (Conservation and Management) Rules, 2010 were framed by the

Government of India based on the recommendations of Ramsar Convention.

Page 98: UPSC 2019 PAPER...UPSC 2019 PAPER SET- D 1. With reference to Asian Infrastructure Investment Bank (AIIB), consider the following statements 1. AIIB has more than 80 member nations

3. The Wetlands (Conservation and Management) Rules, 2010 also encompass the drainage

area or catchment regions of the wetlands as determined by the authority.

Which of the statements given above is / are correct?

(a) 1 and 2 only

(b) 2 and 3 only

(c) 3 only

(d) 1, 2 and 3

Sol. 100 (c) 3 only

Source of 1st and 2nd Statement: PrepMate Environment & Biodiversity Book, Chapter 22,

Page 319

Statement 1 is incorrect.

Statement 2 is also incorrect. The Wetlands (Conservation and Management) Rules are

framed in 2010 and Ramsar Convention came into existence in the year 1971. Moreover,

the agenda of the convention does not mandate nations to frame rules for protection of

wetlands.

Once we rule out these statements, clearly option (c) is the right answer.